Which one of the following is an acceptable list of the volunteers and the tasks each demonstrates, in order from the...

maonuo on September 10, 2020

Oct2002-S2-Q8

Could you please help with the game setup?

Reply
Create a free account to read and take part in forum discussions.

Already have an account? log in

Victoria on September 17, 2020

Hi @maonuo,

Happy to help!

We know that there will be six tasks demonstrated at a farm exhibition: H, M, P, S, T, and W.

Each task will be demonstrated exactly once and no two tasks will be demonstrated concurrently. This means we need to make a sequence. However, we also need to schedule the volunteers so we are dealing with a Multi-Linear game.

Each task will be demonstrated by one of three volunteers: F, G, and L.

Each volunteer will demonstrate exactly two tasks.

T: _ _ _ _ _ _
V: _ _ _ _ _ _

Now let's go through our rules.

Rule 1 - F demonstrates exactly one task before G demonstrates any of their tasks.

> G
F > G
> F

Rule 2 - F performs neither the first nor the last demonstration.

We know that F must perform exactly one task before G demonstrates any tasks. As F cannot perform the first demonstration, L must perform the first demonstration.

Rule 3 - G demonstrates neither H nor M

G --> Not H and Not M
H or M --> Not G

Rule 4 - L demonstrates neither H nor T

L --> Not H and Not T
H or T --> Not L

We know from Rules 3 and 4 that neither G nor L can perform H. Therefore, F must perform H and H cannot be performed last.

Rule 5 - M is the next task demonstrated after T is demonstrated.

|TM|

Overall Set-up

T: _ _ _ _ _ _
V: L _ _ _ _ _

> G
F > G
> F

H = F

Now let's address the question stem. We are looking for the answer choice which outlines an acceptable list of volunteers and the tasks which each demonstrates. Therefore, we are looking for the answer choice which could be true. Each incorrect answer choice cannot be true.

We can eliminate answer choice (A) for violating Rule 2 because F performs the first demonstration here.

We can eliminate answer choice (B) for violating Rule 1. We know that F demonstrates EXACTLY ONE task before G demonstrates any tasks. Here, F demonstrates two tasks prior to G demonstrating any of their tasks.

We can eliminate answer choice (D) for violating Rule 3 as G demonstrates H and we know that F must demonstrate H.

Finally, we can eliminate answer choice (E) for violating Rule 5. Here, S is the next task demonstrated after T is demonstrated. Rule 5 tells us that M is the next task demonstrated after T is demonstrated.

This means that answer choice (C) must be our correct answer, but let's double check.

T: P S T M H W
V: L F G L F G

Notice that this meets all of the conditions:

1) F demonstrates S before G demonstrates T and W.
2) F performs neither the first nor the last demonstration.
3) G demonstrates T and W.
4) L demonstrates P and M.
5) L demonstrates M right after G demonstrates T.

Hope this helps! Please let us know if you have any further questions.